LSAT and Law School Admissions Forum

Get expert LSAT preparation and law school admissions advice from PowerScore Test Preparation.

User avatar
 Dave Killoran
PowerScore Staff
  • PowerScore Staff
  • Posts: 5853
  • Joined: Mar 25, 2011
|
#59624
Complete Question Explanation
(The complete setup for this game can be found here: lsat/viewtopic.php?t=15933)

The correct answer choice is (D)

The question stem creates a horizontal HS block:

pt44_o04_g2_q11a.png
This block must be placed on either Monday and Tuesday, or Tuesday and Wednesday. Consequently, the LP vertical block must be placed on Monday or Wednesday. This deduction is sufficient to show that P cannot be placed on Tuesday, and so answer choice (D) is correct. However, in the interests of fully understanding the relationships at work in this question, let’s continue our analysis. So far we have two blocks, and since HS is a horizontal block, we will show the other spaces with H and S:

pt44_o04_g2_q11b.png
The remaining two variables are G and K. Because the second rule specifies that G and H cannot be placed on the same day, we can infer that G must placed with S. K must then be placed with H:

pt44_o04_g2_q11c.png
Because of the fortuitous arrangement of the variables, the blocks can be placed in any order.
Answer choices (A), (B), (C) and (E): Each of these answer choices could be true, as proven by the following hypothetical:

pt44_o04_g2_q11d.png
Answer choice (D): This is the correct answer choice. As discussed above, the LP vertical block must be placed on either Monday or Wednesday.
You do not have the required permissions to view the files attached to this post.

Get the most out of your LSAT Prep Plus subscription.

Analyze and track your performance with our Testing and Analytics Package.